Difference between revisions of "2023 AMC 10B Problems/Problem 18"

(Created page with "The answers will be uploaded soon - MRP")
 
Line 1: Line 1:
The answers will be uploaded soon
+
== Solution ==
- MRP
+
<math>I.</math>  Try <math>a=3,b=5 => c = 17\cdot15</math> which makes <math>\textbf{I}</math> false.
 +
At this point, we can rule out answer A,B,C.
 +
 
 +
<math>II.</math> A => B or C. equiv. ~B AND ~C => ~A.
 +
Let a = 14, b=15 (statisfying ~B and ~C). => C = 2*210. which is ~A.
 +
 
 +
<math>II</math> is true.
 +
 
 +
So the answer is E.
 +
<math>\boxed{\textbf{(E) } II \text{ and } III \text{only}.}</math>
 +
~Technodoggo

Revision as of 15:23, 15 November 2023

Solution

$I.$ Try $a=3,b=5 => c = 17\cdot15$ which makes $\textbf{I}$ false. At this point, we can rule out answer A,B,C.

$II.$ A => B or C. equiv. ~B AND ~C => ~A. Let a = 14, b=15 (statisfying ~B and ~C). => C = 2*210. which is ~A.

$II$ is true.

So the answer is E. $\boxed{\textbf{(E) } II \text{ and } III \text{only}.}$ ~Technodoggo